An otherwise healthy 1-month-old female presents with lesions on the face, scalp, and chest

Article Type
Changed
Tue, 06/15/2021 - 09:30

A potassium hydroxide preparation (KOH) from skin scrapings from the scalp lesions demonstrated no fungal elements. Further laboratory work up revealed a normal blood cell count, normal liver enzymes, an antinuclear antibody (ANA) titer of less than 1:80, a positive anti–Sjögren’s syndrome type B (SSB) antibody but negative anti–Sjögren’s syndrome type A (SSA) antibody and anti-U1RNP antibody. An electrocardiogram revealed no abnormalities. Liver function tests were normal. The complete blood count showed mild thrombocytopenia. Given the typical skin lesions and the positive SSB test and associated thrombocytopenia, the baby was diagnosed with neonatal lupus erythematosus.

Dr. Catalina Matiz, a pediatric dermatologist at Southern California Permanente Medical Group, San Diego
Dr. Catalina Matiz

Because of the diagnosis of neonatal lupus the mother was also tested and was found to have an elevated ANA of 1:640, positive SSB and antiphospholipid antibodies. The mother was healthy and her review of systems was negative for any collagen vascular disease–related symptoms.
 

Discussion

Neonatal lupus erythematosus (NLE) is a rare form of systemic lupus erythematosus (SLE) believed to be caused by transplacental transfer of anti-Ro (Sjögren’s syndrome antigen A, SSA), or, less commonly, anti-La (Sjögren’s syndrome antigen B, SSB) from mothers who are positive for these antibodies. Approximately 95% of NLE is associated with maternal anti-SSA; of these cases, 40% are also associated with maternal anti-SSB.1 Only about 2% of children of mothers who have anti-SSA or anti-SSB develop NLE, a finding that has led some researchers to postulate that maternal factors, fetal genetic factors, and environmental factors determine which children of anti-SSA or SSB positive mothers develop NLE.

A recent review found no association between the development of NLE and fetal birth weight, prematurity, or age.3 Over half of mothers of children who develop NLE are asymptomatic at the time of diagnosis of the neonate,3 though many become symptomatic in following years. Of mothers who are symptomatic, SLE and undifferentiated autoimmune syndrome are the most common diagnoses, though NLE has been rarely reported in the offspring of mothers with Sjögren’s syndrome, rheumatoid arthritis, and psoriasis.4,5

Fetal genetics are not an absolute determinant of development of NLE, as discordance in the development of NLE in twins has been reported. However, certain genetic relationships have been established. Fetal mutations in tumor necrosis factor–alpha appear to increase the likelihood of cutaneous manifestations. Mutations in transforming growth factor beta appear to increase the likelihood of cardiac manifestations, and experiments in cultured mouse cardiocytes have shown anti-SSB antibodies to impair macrophage phagocytosis of apoptotic cells in the developing fetal heart. These observations taken together suggest a fibroblast-mediated response to unphagocytosed cardiocyte debris may account for conduction abnormalities in neonates with NLE-induced heart block.6

Cutaneous disease in NLE is possible at birth, but more skin findings develop upon exposure to the sun. Nearly 80% of neonates affected by NLE develop cutaneous manifestations in the first few months of life. The head, neck, and extensor surfaces of the arms are most commonly affected, presumably because they are most likely to be exposed to the sun. Erythematous, annular, or discoid lesions are most common, and periorbital erythema with or without scale (“raccoon eyes”) should prompt consideration of NLE. However, annular, or discoid lesions are sometimes not present in NLE; telangiectasias, bullae, atrophic divots (“ice-pick scars”) or ulcerations may be seen instead. Lesions in the genital area have been described in fewer than 5% of patients with NLE.

The differential diagnosis of annular, scaly lesions in neonates includes annular erythema of infancy, tinea corporis, and seborrheic dermatitis. Annular erythema of infancy is a rare skin condition characterized by a cyclical eruption of erythematous annular lesions with minimal scaling which resolve spontaneously within a few weeks to months without leaving scaring or pigment changes. There is no treatment needed as the lesions self-resolve.7 Acute urticaria can sometimes appear similar to NLE but these are not scaly and also the lesions will disappear within 24-36 hours, compared with NLE lesions, which may take weeks to months to go away. Seborrheic dermatitis is a common skin condition seen in newborns with in the first few weeks of life and can present as scaly annular erythematous plaques on the face, scalp, torso, and the diaper area. Seborrheic dermatitis usually responds well to a combination of an antiyeast cream and a low-potency topical corticosteroid medication.

Ayan Kusari, University of California, San Francisco
Ayan Kusari

When NLE is suspected, diagnostic testing for lupus antibodies (anti-SSA, anti-SSB, and anti-U1RNP) in both maternal and neonatal serum should be undertaken. The presence of a characteristic rash plus maternal or neonatal antibodies is sufficient to make the diagnosis. If the rash is less characteristic, a biopsy showing an interface dermatitis can help solidify the diagnosis. Neonates with cutaneous manifestations of lupus may also have systemic disease. The most common and serious complication is heart block, whose pathophysiology is described above. Neonates with evidence of first-, second-, or third-degree heart block should be referred to a pediatric cardiologist for careful monitoring and management. Hepatic involvement has been reported, but is usually mild. Hematologic abnormalities have also been described that include anemia, neutropenia, and thrombocytopenia, which resolve by 9 months of age. Central nervous system involvement may rarely occur. The mainstay of treatment for the rash in NLE is diligent sun avoidance and sun protection. Topical corticosteroids may be used, but are not needed as the rash typically resolves by 9 months to 1 year without treatment. Mothers who have one child with NLE should be advised that they are more likely to have another with NLE – the risk is as high as 30%-40% in the second child. Hydroxychloroquine taken during subsequent pregnancies can reduce the incidence of cardiac complications,8 as can the so-called “triple therapy” of plasmapheresis, steroids, and IVIg.9

The cutaneous manifestations of NLE are usually self-limiting. However, they can serve as important clues that can prompt diagnosis of SLE in the mother, investigation of cardiac complications in the infant, and appropriate preventative care in future pregnancies.

Dr. Matiz is with the department of dermatology, Southern California Permanente Medical Group, San Diego. Mr. Kusari is with the department of dermatology, University of California, San Francisco.

References

1. Moretti D et al. Int J Dermatol. 2014;53(12):1508-12.

2. Buyon JP et al. Nature Clin Prac Rheum. 2009;5(3):139-48.

3. Li Y-Q et al. Int J Rheum Dis. 2015;18(7):761-7.

4. Rivera TL et al. Annals Rheum Dis. 2009;68(6):828-35.

5. Li L et al. Zhonghua er ke za zhi 2011;49(2):146-50.

6. Izmirly PM et al. Clin Rheumatol. 2011;30(12):1641-5.

7. Toledo-Alberola F and Betlloch-Mas I. Actas Dermosifiliogr. 2010 Jul;101(6):473-84.

8. Izmirly PM et al. Circulation. 2012;126(1):76-82.

9. Martinez-Sanchez N et al. Autoimmun Rev. 2015;14(5):423-8.

Publications
Topics
Sections

A potassium hydroxide preparation (KOH) from skin scrapings from the scalp lesions demonstrated no fungal elements. Further laboratory work up revealed a normal blood cell count, normal liver enzymes, an antinuclear antibody (ANA) titer of less than 1:80, a positive anti–Sjögren’s syndrome type B (SSB) antibody but negative anti–Sjögren’s syndrome type A (SSA) antibody and anti-U1RNP antibody. An electrocardiogram revealed no abnormalities. Liver function tests were normal. The complete blood count showed mild thrombocytopenia. Given the typical skin lesions and the positive SSB test and associated thrombocytopenia, the baby was diagnosed with neonatal lupus erythematosus.

Dr. Catalina Matiz, a pediatric dermatologist at Southern California Permanente Medical Group, San Diego
Dr. Catalina Matiz

Because of the diagnosis of neonatal lupus the mother was also tested and was found to have an elevated ANA of 1:640, positive SSB and antiphospholipid antibodies. The mother was healthy and her review of systems was negative for any collagen vascular disease–related symptoms.
 

Discussion

Neonatal lupus erythematosus (NLE) is a rare form of systemic lupus erythematosus (SLE) believed to be caused by transplacental transfer of anti-Ro (Sjögren’s syndrome antigen A, SSA), or, less commonly, anti-La (Sjögren’s syndrome antigen B, SSB) from mothers who are positive for these antibodies. Approximately 95% of NLE is associated with maternal anti-SSA; of these cases, 40% are also associated with maternal anti-SSB.1 Only about 2% of children of mothers who have anti-SSA or anti-SSB develop NLE, a finding that has led some researchers to postulate that maternal factors, fetal genetic factors, and environmental factors determine which children of anti-SSA or SSB positive mothers develop NLE.

A recent review found no association between the development of NLE and fetal birth weight, prematurity, or age.3 Over half of mothers of children who develop NLE are asymptomatic at the time of diagnosis of the neonate,3 though many become symptomatic in following years. Of mothers who are symptomatic, SLE and undifferentiated autoimmune syndrome are the most common diagnoses, though NLE has been rarely reported in the offspring of mothers with Sjögren’s syndrome, rheumatoid arthritis, and psoriasis.4,5

Fetal genetics are not an absolute determinant of development of NLE, as discordance in the development of NLE in twins has been reported. However, certain genetic relationships have been established. Fetal mutations in tumor necrosis factor–alpha appear to increase the likelihood of cutaneous manifestations. Mutations in transforming growth factor beta appear to increase the likelihood of cardiac manifestations, and experiments in cultured mouse cardiocytes have shown anti-SSB antibodies to impair macrophage phagocytosis of apoptotic cells in the developing fetal heart. These observations taken together suggest a fibroblast-mediated response to unphagocytosed cardiocyte debris may account for conduction abnormalities in neonates with NLE-induced heart block.6

Cutaneous disease in NLE is possible at birth, but more skin findings develop upon exposure to the sun. Nearly 80% of neonates affected by NLE develop cutaneous manifestations in the first few months of life. The head, neck, and extensor surfaces of the arms are most commonly affected, presumably because they are most likely to be exposed to the sun. Erythematous, annular, or discoid lesions are most common, and periorbital erythema with or without scale (“raccoon eyes”) should prompt consideration of NLE. However, annular, or discoid lesions are sometimes not present in NLE; telangiectasias, bullae, atrophic divots (“ice-pick scars”) or ulcerations may be seen instead. Lesions in the genital area have been described in fewer than 5% of patients with NLE.

The differential diagnosis of annular, scaly lesions in neonates includes annular erythema of infancy, tinea corporis, and seborrheic dermatitis. Annular erythema of infancy is a rare skin condition characterized by a cyclical eruption of erythematous annular lesions with minimal scaling which resolve spontaneously within a few weeks to months without leaving scaring or pigment changes. There is no treatment needed as the lesions self-resolve.7 Acute urticaria can sometimes appear similar to NLE but these are not scaly and also the lesions will disappear within 24-36 hours, compared with NLE lesions, which may take weeks to months to go away. Seborrheic dermatitis is a common skin condition seen in newborns with in the first few weeks of life and can present as scaly annular erythematous plaques on the face, scalp, torso, and the diaper area. Seborrheic dermatitis usually responds well to a combination of an antiyeast cream and a low-potency topical corticosteroid medication.

Ayan Kusari, University of California, San Francisco
Ayan Kusari

When NLE is suspected, diagnostic testing for lupus antibodies (anti-SSA, anti-SSB, and anti-U1RNP) in both maternal and neonatal serum should be undertaken. The presence of a characteristic rash plus maternal or neonatal antibodies is sufficient to make the diagnosis. If the rash is less characteristic, a biopsy showing an interface dermatitis can help solidify the diagnosis. Neonates with cutaneous manifestations of lupus may also have systemic disease. The most common and serious complication is heart block, whose pathophysiology is described above. Neonates with evidence of first-, second-, or third-degree heart block should be referred to a pediatric cardiologist for careful monitoring and management. Hepatic involvement has been reported, but is usually mild. Hematologic abnormalities have also been described that include anemia, neutropenia, and thrombocytopenia, which resolve by 9 months of age. Central nervous system involvement may rarely occur. The mainstay of treatment for the rash in NLE is diligent sun avoidance and sun protection. Topical corticosteroids may be used, but are not needed as the rash typically resolves by 9 months to 1 year without treatment. Mothers who have one child with NLE should be advised that they are more likely to have another with NLE – the risk is as high as 30%-40% in the second child. Hydroxychloroquine taken during subsequent pregnancies can reduce the incidence of cardiac complications,8 as can the so-called “triple therapy” of plasmapheresis, steroids, and IVIg.9

The cutaneous manifestations of NLE are usually self-limiting. However, they can serve as important clues that can prompt diagnosis of SLE in the mother, investigation of cardiac complications in the infant, and appropriate preventative care in future pregnancies.

Dr. Matiz is with the department of dermatology, Southern California Permanente Medical Group, San Diego. Mr. Kusari is with the department of dermatology, University of California, San Francisco.

References

1. Moretti D et al. Int J Dermatol. 2014;53(12):1508-12.

2. Buyon JP et al. Nature Clin Prac Rheum. 2009;5(3):139-48.

3. Li Y-Q et al. Int J Rheum Dis. 2015;18(7):761-7.

4. Rivera TL et al. Annals Rheum Dis. 2009;68(6):828-35.

5. Li L et al. Zhonghua er ke za zhi 2011;49(2):146-50.

6. Izmirly PM et al. Clin Rheumatol. 2011;30(12):1641-5.

7. Toledo-Alberola F and Betlloch-Mas I. Actas Dermosifiliogr. 2010 Jul;101(6):473-84.

8. Izmirly PM et al. Circulation. 2012;126(1):76-82.

9. Martinez-Sanchez N et al. Autoimmun Rev. 2015;14(5):423-8.

A potassium hydroxide preparation (KOH) from skin scrapings from the scalp lesions demonstrated no fungal elements. Further laboratory work up revealed a normal blood cell count, normal liver enzymes, an antinuclear antibody (ANA) titer of less than 1:80, a positive anti–Sjögren’s syndrome type B (SSB) antibody but negative anti–Sjögren’s syndrome type A (SSA) antibody and anti-U1RNP antibody. An electrocardiogram revealed no abnormalities. Liver function tests were normal. The complete blood count showed mild thrombocytopenia. Given the typical skin lesions and the positive SSB test and associated thrombocytopenia, the baby was diagnosed with neonatal lupus erythematosus.

Dr. Catalina Matiz, a pediatric dermatologist at Southern California Permanente Medical Group, San Diego
Dr. Catalina Matiz

Because of the diagnosis of neonatal lupus the mother was also tested and was found to have an elevated ANA of 1:640, positive SSB and antiphospholipid antibodies. The mother was healthy and her review of systems was negative for any collagen vascular disease–related symptoms.
 

Discussion

Neonatal lupus erythematosus (NLE) is a rare form of systemic lupus erythematosus (SLE) believed to be caused by transplacental transfer of anti-Ro (Sjögren’s syndrome antigen A, SSA), or, less commonly, anti-La (Sjögren’s syndrome antigen B, SSB) from mothers who are positive for these antibodies. Approximately 95% of NLE is associated with maternal anti-SSA; of these cases, 40% are also associated with maternal anti-SSB.1 Only about 2% of children of mothers who have anti-SSA or anti-SSB develop NLE, a finding that has led some researchers to postulate that maternal factors, fetal genetic factors, and environmental factors determine which children of anti-SSA or SSB positive mothers develop NLE.

A recent review found no association between the development of NLE and fetal birth weight, prematurity, or age.3 Over half of mothers of children who develop NLE are asymptomatic at the time of diagnosis of the neonate,3 though many become symptomatic in following years. Of mothers who are symptomatic, SLE and undifferentiated autoimmune syndrome are the most common diagnoses, though NLE has been rarely reported in the offspring of mothers with Sjögren’s syndrome, rheumatoid arthritis, and psoriasis.4,5

Fetal genetics are not an absolute determinant of development of NLE, as discordance in the development of NLE in twins has been reported. However, certain genetic relationships have been established. Fetal mutations in tumor necrosis factor–alpha appear to increase the likelihood of cutaneous manifestations. Mutations in transforming growth factor beta appear to increase the likelihood of cardiac manifestations, and experiments in cultured mouse cardiocytes have shown anti-SSB antibodies to impair macrophage phagocytosis of apoptotic cells in the developing fetal heart. These observations taken together suggest a fibroblast-mediated response to unphagocytosed cardiocyte debris may account for conduction abnormalities in neonates with NLE-induced heart block.6

Cutaneous disease in NLE is possible at birth, but more skin findings develop upon exposure to the sun. Nearly 80% of neonates affected by NLE develop cutaneous manifestations in the first few months of life. The head, neck, and extensor surfaces of the arms are most commonly affected, presumably because they are most likely to be exposed to the sun. Erythematous, annular, or discoid lesions are most common, and periorbital erythema with or without scale (“raccoon eyes”) should prompt consideration of NLE. However, annular, or discoid lesions are sometimes not present in NLE; telangiectasias, bullae, atrophic divots (“ice-pick scars”) or ulcerations may be seen instead. Lesions in the genital area have been described in fewer than 5% of patients with NLE.

The differential diagnosis of annular, scaly lesions in neonates includes annular erythema of infancy, tinea corporis, and seborrheic dermatitis. Annular erythema of infancy is a rare skin condition characterized by a cyclical eruption of erythematous annular lesions with minimal scaling which resolve spontaneously within a few weeks to months without leaving scaring or pigment changes. There is no treatment needed as the lesions self-resolve.7 Acute urticaria can sometimes appear similar to NLE but these are not scaly and also the lesions will disappear within 24-36 hours, compared with NLE lesions, which may take weeks to months to go away. Seborrheic dermatitis is a common skin condition seen in newborns with in the first few weeks of life and can present as scaly annular erythematous plaques on the face, scalp, torso, and the diaper area. Seborrheic dermatitis usually responds well to a combination of an antiyeast cream and a low-potency topical corticosteroid medication.

Ayan Kusari, University of California, San Francisco
Ayan Kusari

When NLE is suspected, diagnostic testing for lupus antibodies (anti-SSA, anti-SSB, and anti-U1RNP) in both maternal and neonatal serum should be undertaken. The presence of a characteristic rash plus maternal or neonatal antibodies is sufficient to make the diagnosis. If the rash is less characteristic, a biopsy showing an interface dermatitis can help solidify the diagnosis. Neonates with cutaneous manifestations of lupus may also have systemic disease. The most common and serious complication is heart block, whose pathophysiology is described above. Neonates with evidence of first-, second-, or third-degree heart block should be referred to a pediatric cardiologist for careful monitoring and management. Hepatic involvement has been reported, but is usually mild. Hematologic abnormalities have also been described that include anemia, neutropenia, and thrombocytopenia, which resolve by 9 months of age. Central nervous system involvement may rarely occur. The mainstay of treatment for the rash in NLE is diligent sun avoidance and sun protection. Topical corticosteroids may be used, but are not needed as the rash typically resolves by 9 months to 1 year without treatment. Mothers who have one child with NLE should be advised that they are more likely to have another with NLE – the risk is as high as 30%-40% in the second child. Hydroxychloroquine taken during subsequent pregnancies can reduce the incidence of cardiac complications,8 as can the so-called “triple therapy” of plasmapheresis, steroids, and IVIg.9

The cutaneous manifestations of NLE are usually self-limiting. However, they can serve as important clues that can prompt diagnosis of SLE in the mother, investigation of cardiac complications in the infant, and appropriate preventative care in future pregnancies.

Dr. Matiz is with the department of dermatology, Southern California Permanente Medical Group, San Diego. Mr. Kusari is with the department of dermatology, University of California, San Francisco.

References

1. Moretti D et al. Int J Dermatol. 2014;53(12):1508-12.

2. Buyon JP et al. Nature Clin Prac Rheum. 2009;5(3):139-48.

3. Li Y-Q et al. Int J Rheum Dis. 2015;18(7):761-7.

4. Rivera TL et al. Annals Rheum Dis. 2009;68(6):828-35.

5. Li L et al. Zhonghua er ke za zhi 2011;49(2):146-50.

6. Izmirly PM et al. Clin Rheumatol. 2011;30(12):1641-5.

7. Toledo-Alberola F and Betlloch-Mas I. Actas Dermosifiliogr. 2010 Jul;101(6):473-84.

8. Izmirly PM et al. Circulation. 2012;126(1):76-82.

9. Martinez-Sanchez N et al. Autoimmun Rev. 2015;14(5):423-8.

Publications
Publications
Topics
Article Type
Sections
Questionnaire Body

A 1-month-old, full-term female, born via normal vaginal delivery, presented to the dermatology clinic with a 3-week history of recurrent skin lesions on the scalp, face, and chest. The mother has been treating the lesions with breast milk and most recently with clotrimazole cream without resolution.  


The mother of the baby is a healthy 32-year-old female with no past medical history. She had adequate prenatal care, and all the prenatal infectious and genetic tests were normal. The baby has been healthy and growing well. There is no history of associated fevers, chills, or any other symptoms. The family took no recent trips, and the parents are not affected. There are no other children at home and they have a cat and a dog. The family history is noncontributory.  
On physical examination the baby was not in acute distress and her vital signs were normal. On skin examination she had several erythematous annular plaques and patches on the face, scalp, and upper chest (Fig. 1). There was no liver or spleen enlargement and no lymphadenopathy was palpated on exam. 

Disallow All Ads
Content Gating
No Gating (article Unlocked/Free)
Alternative CME
Disqus Comments
Default
Use ProPublica
Hide sidebar & use full width
render the right sidebar.
Conference Recap Checkbox
Not Conference Recap
Clinical Edge
Display the Slideshow in this Article
Medscape Article
Display survey writer
Reuters content
Disable Inline Native ads
WebMD Article

Pediatric Dermatology Consult - June 2018

Article Type
Changed
Fri, 01/18/2019 - 17:42

 

The patient was diagnosed with granuloma annulare on the basis of history and clinical exam. A potassium hydroxide prep of skin scrapings was performed to rule out tinea corporis, and did not show evidence of fungal elements. The patient was treated with topical betamethasone with partial improvement.

First described as a “ringed eruption of the fingers” by Thomas Colcott Fox in 1895, granuloma annulare (GA) is a relatively common, benign, and self-limited condition whose precise etiology remains unclear. It is characterized commonly by pink to violaceous aciform or annular plaques on clinical examination. In some cases of GA, annular lesions are not present, or may be formed of grouped papules.

Pink annular plaque over left first toe.
Courtesy Dr. Catalina Matiz
Pink annular plaque over left first toe.
GA is a relatively common condition, accounting for up to 0.4% of dermatologists’ new patient visits according to one estimate.1 GA is most common in children and young adults under 30 years, but has a bimodal distribution with a second, smaller peak of incidence in older adults aged 50-70 years.2 In the younger group, localized GA is the most common presentation, whereas generalized GA is relatively more common in the older group. Across groups, there is a female predilection; approximately two-thirds of patients with GA are female.2

GA is characterized histologically by patchy interstitial lymphocytes and histiocytes palisading around mucin. Deep GA, an unusual subtype observed only in children, features a fibrin rather than a mucin core. This granulomatous picture is consistent with a Th1-mediated inflammatory process, and indeed, macrophage tumor necrosis factor production, as well as interleukin-2 and interferon-gamma production have been observed in GA. The reason for this exaggerated Th1 response is unknown, although in susceptible individuals trauma3 (an example of the Koebner phenomenon), arthropod assault,4 and herpes simplex infection5 (an example of Wolf isotopic response) all have been observed to trigger localized and/or generalized GA. Generalized GA has been associated with hyperlipidemia and the human leukocyte antigen–BW35 allele. GA has been described as a paraneoplastic eruption; atypical features such as associated pain or appearance in an uncharacteristic location often are present in such cases.6,7

Pink round and annular plaques over shins
Courtesy Dr. Catalina Matiz
Pink round and annular plaques over shins
Although annular, erythematous plaques or patches on the dorsal hands or feet are the most common clinical manifestation of GA, a range of presentations exist. The lesions may be flesh-colored, faint pink, or violaceous. The lesions may include papules or plaques; in some cases, annular lesions may not be present. In children, subcutaneous or deep-dermal GA may resemble rheumatoid nodules. Some papular lesions may have central umbilication or papulation because of transepidermal elimination of mucin or collagen; in these cases, GA may be mistaken for molluscum contagiosum. However, despite these variations in clinical appearance, attention should be paid to symptomatology as GA is not pruritic.

Diagnosis of typical GA is clinical. If unusual features make you suspect tinea, leprosy, mycosis fungoides, or other annular lesions, then biopsy showing features typical of GA can reveal the correct diagnosis. Biopsy also can help to distinguish papular GA from warts or molluscum contagiosum. If extensive GA are present, then serum lipid testing for hypercholesterolemia or hypertriglyceridemia should be considered.

Other annular and raised lesions are on the differential for GA, but careful attention to the patient’s history and examination can clarify the diagnosis. Urticaria multiforme, a variant of annular urticaria, presents with numerous annular and polycyclic wheals, sometimes with central darkening that may be mistaken for necrosis. This patient did not present with polycyclic wheals. Furthermore, the lesions in urticaria multiforme are typically transient, with individual lesions lasting less than 24 hours, which was not the case with this patient. Wells syndrome, also known as eosinophilic cellulitis, is a condition marked by recurrent episodes of pruritus followed by appearance of edematous, painful, indurated, or edematous papules or plaques, although bullae and vesicles also may be present. The face and extremities are frequently involved and spontaneous resolution typically occurs in 2 months. Annular lesions are possible but papules, plaques, and nodules are more common in Wells syndrome. Annular elastolytic giant cell granuloma (AEGCG), also known as actinic granuloma and Miescher granuloma of the face, is an entity characterized clinically by chronic, persistent, sun-distributed, annular plaques typically seen in older women with significant sun damage. AEGCG is considered by some to be a variant of GA, but if this is the case, it is a distinct subtype with different epidemiologic, clinical, and histopathologic characteristics from GA. Interstitial granulomatous dermatitis is histologically and clinically distinct from GA, presenting as subtly erythematous cords or extensive annular or serpiginous plaques in the axilla, groin, buttocks, or chest, typically in adult patients with rheumatoid arthritis, reactive arthritis, psoriatic arthritis, or ankylosing spondylitis. Tinea corporis, the clinical manifestation of cutaneous dermatophyte infection, may be mistaken for granuloma annulare. However, tinea corporis lesions are scaly, whereas GA does not scale. Histologic examination of tinea corporis reveals hyphae, which are not present in GA.

Ayan Kusari, University of California, San Francisco
Ayan Kusari
GA lesions resolve spontaneously in a matter of months to years. Recurrence of lesions at previous sites is common, but recurrent lesions typically clear more quickly than initial lesions. As GA is not a dangerous condition, and as lesions are typically asymptomatic, reassurance and watchful waiting are the mainstays of treatment for most patients. Topical corticosteroids may be beneficial for patients with superficial lesions, whereas intralesional corticosteroids have been evaluated for deeper and thicker lesions. For patients with more extensive or distressing disease, UVA phototherapy appears to be effective for some patients. Treatment is often unsatisfying, and as a result a variety of treatments have been evaluated for treatment of GA, including surgical excision, cryotherapy, electrodesiccation, pulsed-dye laser, dapsone, niacinamide/nicotinamide, chloroquine, and isotretinoin, with varying results.8,9 However, high-quality data supporting such treatments are lacking.

GA is a relatively common, idiopathic, benign skin disease with numerous annular and papular mimics. Absence of scale, pain, and significant pruritus are important clues to the diagnosis, and biopsy can be helpful when the diagnosis is unclear. Treatment, although not necessary, may be offered using any of a number of modalities. The most consistent and effective healer of GA, however, is time.
Dr. Catalina Matiz, a pediatric dermatologist at Southern California Permanente Medical Group, San Diego
Dr. Catalina Matiz
Mr. Kusari is with the division of pediatric and adolescent dermatology, Rady Children’s Hospital, San Diego, and the departments of dermatology and pediatrics, University of California, San Diego. Dr. Matiz is a pediatric dermatologist at Southern California Permanente Medical Group, San Diego. They reported having no conflicts of interest.

References

1. J Am Acad Dermatol. 1980 Sep;3(3):217-30.

2. J Am Acad Dermatol. 2016 Sep;75(3):457-65.

3. Dermatol Online J. 2013 Dec 16;19(12):20719.

4. Acta Derm Venereol. 2008;88(5):519-20.

5. J Cutan Med Surg. 2014 Nov;18(6):413-9.

6. South Med J. 1997 Oct;90(10):1056-9.

7. Am J Dermatopathol. 2003 Apr;25(2):113-6.

8. Am J Clin Dermatol. 2013 Aug;14(4):279-90.

9. Br J Dermatol. 1994 Apr;130(4):494-7.

Publications
Topics
Sections

 

The patient was diagnosed with granuloma annulare on the basis of history and clinical exam. A potassium hydroxide prep of skin scrapings was performed to rule out tinea corporis, and did not show evidence of fungal elements. The patient was treated with topical betamethasone with partial improvement.

First described as a “ringed eruption of the fingers” by Thomas Colcott Fox in 1895, granuloma annulare (GA) is a relatively common, benign, and self-limited condition whose precise etiology remains unclear. It is characterized commonly by pink to violaceous aciform or annular plaques on clinical examination. In some cases of GA, annular lesions are not present, or may be formed of grouped papules.

Pink annular plaque over left first toe.
Courtesy Dr. Catalina Matiz
Pink annular plaque over left first toe.
GA is a relatively common condition, accounting for up to 0.4% of dermatologists’ new patient visits according to one estimate.1 GA is most common in children and young adults under 30 years, but has a bimodal distribution with a second, smaller peak of incidence in older adults aged 50-70 years.2 In the younger group, localized GA is the most common presentation, whereas generalized GA is relatively more common in the older group. Across groups, there is a female predilection; approximately two-thirds of patients with GA are female.2

GA is characterized histologically by patchy interstitial lymphocytes and histiocytes palisading around mucin. Deep GA, an unusual subtype observed only in children, features a fibrin rather than a mucin core. This granulomatous picture is consistent with a Th1-mediated inflammatory process, and indeed, macrophage tumor necrosis factor production, as well as interleukin-2 and interferon-gamma production have been observed in GA. The reason for this exaggerated Th1 response is unknown, although in susceptible individuals trauma3 (an example of the Koebner phenomenon), arthropod assault,4 and herpes simplex infection5 (an example of Wolf isotopic response) all have been observed to trigger localized and/or generalized GA. Generalized GA has been associated with hyperlipidemia and the human leukocyte antigen–BW35 allele. GA has been described as a paraneoplastic eruption; atypical features such as associated pain or appearance in an uncharacteristic location often are present in such cases.6,7

Pink round and annular plaques over shins
Courtesy Dr. Catalina Matiz
Pink round and annular plaques over shins
Although annular, erythematous plaques or patches on the dorsal hands or feet are the most common clinical manifestation of GA, a range of presentations exist. The lesions may be flesh-colored, faint pink, or violaceous. The lesions may include papules or plaques; in some cases, annular lesions may not be present. In children, subcutaneous or deep-dermal GA may resemble rheumatoid nodules. Some papular lesions may have central umbilication or papulation because of transepidermal elimination of mucin or collagen; in these cases, GA may be mistaken for molluscum contagiosum. However, despite these variations in clinical appearance, attention should be paid to symptomatology as GA is not pruritic.

Diagnosis of typical GA is clinical. If unusual features make you suspect tinea, leprosy, mycosis fungoides, or other annular lesions, then biopsy showing features typical of GA can reveal the correct diagnosis. Biopsy also can help to distinguish papular GA from warts or molluscum contagiosum. If extensive GA are present, then serum lipid testing for hypercholesterolemia or hypertriglyceridemia should be considered.

Other annular and raised lesions are on the differential for GA, but careful attention to the patient’s history and examination can clarify the diagnosis. Urticaria multiforme, a variant of annular urticaria, presents with numerous annular and polycyclic wheals, sometimes with central darkening that may be mistaken for necrosis. This patient did not present with polycyclic wheals. Furthermore, the lesions in urticaria multiforme are typically transient, with individual lesions lasting less than 24 hours, which was not the case with this patient. Wells syndrome, also known as eosinophilic cellulitis, is a condition marked by recurrent episodes of pruritus followed by appearance of edematous, painful, indurated, or edematous papules or plaques, although bullae and vesicles also may be present. The face and extremities are frequently involved and spontaneous resolution typically occurs in 2 months. Annular lesions are possible but papules, plaques, and nodules are more common in Wells syndrome. Annular elastolytic giant cell granuloma (AEGCG), also known as actinic granuloma and Miescher granuloma of the face, is an entity characterized clinically by chronic, persistent, sun-distributed, annular plaques typically seen in older women with significant sun damage. AEGCG is considered by some to be a variant of GA, but if this is the case, it is a distinct subtype with different epidemiologic, clinical, and histopathologic characteristics from GA. Interstitial granulomatous dermatitis is histologically and clinically distinct from GA, presenting as subtly erythematous cords or extensive annular or serpiginous plaques in the axilla, groin, buttocks, or chest, typically in adult patients with rheumatoid arthritis, reactive arthritis, psoriatic arthritis, or ankylosing spondylitis. Tinea corporis, the clinical manifestation of cutaneous dermatophyte infection, may be mistaken for granuloma annulare. However, tinea corporis lesions are scaly, whereas GA does not scale. Histologic examination of tinea corporis reveals hyphae, which are not present in GA.

Ayan Kusari, University of California, San Francisco
Ayan Kusari
GA lesions resolve spontaneously in a matter of months to years. Recurrence of lesions at previous sites is common, but recurrent lesions typically clear more quickly than initial lesions. As GA is not a dangerous condition, and as lesions are typically asymptomatic, reassurance and watchful waiting are the mainstays of treatment for most patients. Topical corticosteroids may be beneficial for patients with superficial lesions, whereas intralesional corticosteroids have been evaluated for deeper and thicker lesions. For patients with more extensive or distressing disease, UVA phototherapy appears to be effective for some patients. Treatment is often unsatisfying, and as a result a variety of treatments have been evaluated for treatment of GA, including surgical excision, cryotherapy, electrodesiccation, pulsed-dye laser, dapsone, niacinamide/nicotinamide, chloroquine, and isotretinoin, with varying results.8,9 However, high-quality data supporting such treatments are lacking.

GA is a relatively common, idiopathic, benign skin disease with numerous annular and papular mimics. Absence of scale, pain, and significant pruritus are important clues to the diagnosis, and biopsy can be helpful when the diagnosis is unclear. Treatment, although not necessary, may be offered using any of a number of modalities. The most consistent and effective healer of GA, however, is time.
Dr. Catalina Matiz, a pediatric dermatologist at Southern California Permanente Medical Group, San Diego
Dr. Catalina Matiz
Mr. Kusari is with the division of pediatric and adolescent dermatology, Rady Children’s Hospital, San Diego, and the departments of dermatology and pediatrics, University of California, San Diego. Dr. Matiz is a pediatric dermatologist at Southern California Permanente Medical Group, San Diego. They reported having no conflicts of interest.

References

1. J Am Acad Dermatol. 1980 Sep;3(3):217-30.

2. J Am Acad Dermatol. 2016 Sep;75(3):457-65.

3. Dermatol Online J. 2013 Dec 16;19(12):20719.

4. Acta Derm Venereol. 2008;88(5):519-20.

5. J Cutan Med Surg. 2014 Nov;18(6):413-9.

6. South Med J. 1997 Oct;90(10):1056-9.

7. Am J Dermatopathol. 2003 Apr;25(2):113-6.

8. Am J Clin Dermatol. 2013 Aug;14(4):279-90.

9. Br J Dermatol. 1994 Apr;130(4):494-7.

 

The patient was diagnosed with granuloma annulare on the basis of history and clinical exam. A potassium hydroxide prep of skin scrapings was performed to rule out tinea corporis, and did not show evidence of fungal elements. The patient was treated with topical betamethasone with partial improvement.

First described as a “ringed eruption of the fingers” by Thomas Colcott Fox in 1895, granuloma annulare (GA) is a relatively common, benign, and self-limited condition whose precise etiology remains unclear. It is characterized commonly by pink to violaceous aciform or annular plaques on clinical examination. In some cases of GA, annular lesions are not present, or may be formed of grouped papules.

Pink annular plaque over left first toe.
Courtesy Dr. Catalina Matiz
Pink annular plaque over left first toe.
GA is a relatively common condition, accounting for up to 0.4% of dermatologists’ new patient visits according to one estimate.1 GA is most common in children and young adults under 30 years, but has a bimodal distribution with a second, smaller peak of incidence in older adults aged 50-70 years.2 In the younger group, localized GA is the most common presentation, whereas generalized GA is relatively more common in the older group. Across groups, there is a female predilection; approximately two-thirds of patients with GA are female.2

GA is characterized histologically by patchy interstitial lymphocytes and histiocytes palisading around mucin. Deep GA, an unusual subtype observed only in children, features a fibrin rather than a mucin core. This granulomatous picture is consistent with a Th1-mediated inflammatory process, and indeed, macrophage tumor necrosis factor production, as well as interleukin-2 and interferon-gamma production have been observed in GA. The reason for this exaggerated Th1 response is unknown, although in susceptible individuals trauma3 (an example of the Koebner phenomenon), arthropod assault,4 and herpes simplex infection5 (an example of Wolf isotopic response) all have been observed to trigger localized and/or generalized GA. Generalized GA has been associated with hyperlipidemia and the human leukocyte antigen–BW35 allele. GA has been described as a paraneoplastic eruption; atypical features such as associated pain or appearance in an uncharacteristic location often are present in such cases.6,7

Pink round and annular plaques over shins
Courtesy Dr. Catalina Matiz
Pink round and annular plaques over shins
Although annular, erythematous plaques or patches on the dorsal hands or feet are the most common clinical manifestation of GA, a range of presentations exist. The lesions may be flesh-colored, faint pink, or violaceous. The lesions may include papules or plaques; in some cases, annular lesions may not be present. In children, subcutaneous or deep-dermal GA may resemble rheumatoid nodules. Some papular lesions may have central umbilication or papulation because of transepidermal elimination of mucin or collagen; in these cases, GA may be mistaken for molluscum contagiosum. However, despite these variations in clinical appearance, attention should be paid to symptomatology as GA is not pruritic.

Diagnosis of typical GA is clinical. If unusual features make you suspect tinea, leprosy, mycosis fungoides, or other annular lesions, then biopsy showing features typical of GA can reveal the correct diagnosis. Biopsy also can help to distinguish papular GA from warts or molluscum contagiosum. If extensive GA are present, then serum lipid testing for hypercholesterolemia or hypertriglyceridemia should be considered.

Other annular and raised lesions are on the differential for GA, but careful attention to the patient’s history and examination can clarify the diagnosis. Urticaria multiforme, a variant of annular urticaria, presents with numerous annular and polycyclic wheals, sometimes with central darkening that may be mistaken for necrosis. This patient did not present with polycyclic wheals. Furthermore, the lesions in urticaria multiforme are typically transient, with individual lesions lasting less than 24 hours, which was not the case with this patient. Wells syndrome, also known as eosinophilic cellulitis, is a condition marked by recurrent episodes of pruritus followed by appearance of edematous, painful, indurated, or edematous papules or plaques, although bullae and vesicles also may be present. The face and extremities are frequently involved and spontaneous resolution typically occurs in 2 months. Annular lesions are possible but papules, plaques, and nodules are more common in Wells syndrome. Annular elastolytic giant cell granuloma (AEGCG), also known as actinic granuloma and Miescher granuloma of the face, is an entity characterized clinically by chronic, persistent, sun-distributed, annular plaques typically seen in older women with significant sun damage. AEGCG is considered by some to be a variant of GA, but if this is the case, it is a distinct subtype with different epidemiologic, clinical, and histopathologic characteristics from GA. Interstitial granulomatous dermatitis is histologically and clinically distinct from GA, presenting as subtly erythematous cords or extensive annular or serpiginous plaques in the axilla, groin, buttocks, or chest, typically in adult patients with rheumatoid arthritis, reactive arthritis, psoriatic arthritis, or ankylosing spondylitis. Tinea corporis, the clinical manifestation of cutaneous dermatophyte infection, may be mistaken for granuloma annulare. However, tinea corporis lesions are scaly, whereas GA does not scale. Histologic examination of tinea corporis reveals hyphae, which are not present in GA.

Ayan Kusari, University of California, San Francisco
Ayan Kusari
GA lesions resolve spontaneously in a matter of months to years. Recurrence of lesions at previous sites is common, but recurrent lesions typically clear more quickly than initial lesions. As GA is not a dangerous condition, and as lesions are typically asymptomatic, reassurance and watchful waiting are the mainstays of treatment for most patients. Topical corticosteroids may be beneficial for patients with superficial lesions, whereas intralesional corticosteroids have been evaluated for deeper and thicker lesions. For patients with more extensive or distressing disease, UVA phototherapy appears to be effective for some patients. Treatment is often unsatisfying, and as a result a variety of treatments have been evaluated for treatment of GA, including surgical excision, cryotherapy, electrodesiccation, pulsed-dye laser, dapsone, niacinamide/nicotinamide, chloroquine, and isotretinoin, with varying results.8,9 However, high-quality data supporting such treatments are lacking.

GA is a relatively common, idiopathic, benign skin disease with numerous annular and papular mimics. Absence of scale, pain, and significant pruritus are important clues to the diagnosis, and biopsy can be helpful when the diagnosis is unclear. Treatment, although not necessary, may be offered using any of a number of modalities. The most consistent and effective healer of GA, however, is time.
Dr. Catalina Matiz, a pediatric dermatologist at Southern California Permanente Medical Group, San Diego
Dr. Catalina Matiz
Mr. Kusari is with the division of pediatric and adolescent dermatology, Rady Children’s Hospital, San Diego, and the departments of dermatology and pediatrics, University of California, San Diego. Dr. Matiz is a pediatric dermatologist at Southern California Permanente Medical Group, San Diego. They reported having no conflicts of interest.

References

1. J Am Acad Dermatol. 1980 Sep;3(3):217-30.

2. J Am Acad Dermatol. 2016 Sep;75(3):457-65.

3. Dermatol Online J. 2013 Dec 16;19(12):20719.

4. Acta Derm Venereol. 2008;88(5):519-20.

5. J Cutan Med Surg. 2014 Nov;18(6):413-9.

6. South Med J. 1997 Oct;90(10):1056-9.

7. Am J Dermatopathol. 2003 Apr;25(2):113-6.

8. Am J Clin Dermatol. 2013 Aug;14(4):279-90.

9. Br J Dermatol. 1994 Apr;130(4):494-7.

Publications
Publications
Topics
Article Type
Sections
Questionnaire Body

A 9-year-old girl presented to the dermatology clinic, referred by her pediatrician, for evaluation of asymptomatic lesions on her shins and feet for 2 months. She started developing one lesion over her right shin with other lesions appearing on the opposite leg a few weeks after, and treated the areas initially with an over-the-counter antifungal cream without improvement. She has been healthy and denied any recent fevers or upper respiratory infections, and said she had not taken any medications or vitamin supplements. She reported camping with her father occasionally but denied any bug or tick bites. No other family members were affected. There was no personal or family history of diabetes mellitus or high cholesterol, and there are no pets at home.

Disallow All Ads
Content Gating
No Gating (article Unlocked/Free)
Alternative CME
Disqus Comments
Default
Use ProPublica

Pediatric Dermatology Consult - February 2018

Article Type
Changed
Fri, 01/18/2019 - 17:25

 

The patient was diagnosed with pityriasis rosea (PR) on the basis of the clinical findings; a biopsy was not performed. The patient’s pruritus was treated with oral hydroxyzine and topical 1% triamcinolone ointment. She experienced itch relief with these treatments. On follow-up at 3 months, the patient’s lesions had mostly resolved with some postinflammatory hyperpigmentation.

Multiple scaly, pink, oval plaques and papules on the 6-year-old female patient's torso are pityriasis rosea lesions.
Courtesy Dr. Catalina Matiz
First described as “roseola annulata” by the English physician Robert Willan in 1798, pityriasis rosea is a benign, self-limited condition whose precise etiology remains unknown. The disease is typically characterized by the eruption of a large, scaly, pink- to salmon-colored “herald patch” followed by the eruption of smaller plaques over a matter of hours or days. Up to 40% of cases may not present with a herald patch. In children, the time lapse between the appearance of the herald patch, when it is present, and the generalized eruption is shorter, about 4 days on average, compared with 14 days in adults.1 The eruption in pediatric PR also resolves more quickly in children – on average, within 16 days – compared with 45 days in adults.1 Most patients experience complete resolution of the lesions within 3 months.

In some patients, flu-like symptoms precede the onset of skin lesions; this has led to speculation regarding a viral etiology for PR. This prodrome, which is present in as many as half of all cases,can include mild headache, low-grade fever, joint aches, or malaise.2 Pityriasis rosea is thought to occur secondary to a systemic activation of human herpesviruses (HHV) 6 and/or HHV-7. Three cases of PR have been reported in the setting of H1N1 influenza virus infection.3 In one small study, HHV-8 was detected by polymerase chain reaction in approximately 20% of biopsy samples of lesional skin in patients with PR.4 However, most research on a viral etiology for pityriasis rosea has focused on HHV-6 and to a lesser extent HHV-7. DNA from both viruses has been isolated from PR lesions, but at varying detection rates.5,6 Furthermore, HHV-7 DNA has been isolated in as many as 14% of normal individuals without pityriasis rosea, suggesting that the presence of this virus on the skin is fairly common.7

Pityriasis rosea occurs in males and females of all ethnicities, with a slight female predominance. It is rare in young children and older adults. Most cases occur in adolescents and in adults in their twenties and early thirties. Cases occur most frequently in fall and spring.8

The herald patch of pityriasis rosea is typically solitary, but cases with multiple herald patches have been described. The herald patch can range in size from 1-10 cm and usually contains the best example of trailing scale – scale seen on the inside edge of the annular lesion. The satellite lesions of pityriasis rosea are typically papules or plaques with a collarette of scale. These lesions usually are oriented along the Langer cleavage lines, giving them a “Christmas tree” configuration when they appear on the posterior trunk.

Ayan Kusari, University of California, San Francisco
Ayan Kusari
PR usually presents on the trunk rather than on the extremities, although cases in which there is greater involvement of the extremities, axillae, and groin have been described.9 Such cases are referred to as pityriasis rosea inversa and account for fewer than 5% of all cases.9 Other unusual presentations of pityriasis rosea, including purpura, hemorrhagic lesions, targetoid lesions, vesicles, and urticated plaques have been described. In such cases, biopsy can be useful, more to exclude other diagnoses than to definitively diagnose pityriasis rosea. Fewer than 20% of patients may develop oral lesions.
 

Mimics

The herald patch of pityriasis rosea can resemble tinea corporis, and if there is any doubt as to the diagnosis, potassium hydroxide examination (also known as a KOH test) and/or fungal culture should be done to rule out a fungal etiology. However, certain features of this case, particularly the subsequent development of satellite lesions, are more consistent with pityriasis rosea.

Secondary syphilis should be considered in patients who are sexually active. The lesions of secondary syphilis are not typically pruritic, and involvement of the palms and soles is common (whereas such involvement is rare in pityriasis rosea).

Like pityriasis rosea, pityriasis lichenoides et varioliformis acuta (PLEVA) is characterized by papular lesions that resolve spontaneously; the lesions of PLEVA usually evolve to vesicular, necrotic, and purpuric papules that take longer to resolve than PR lesions. The lesions of PLEVA are more erythematous, pustular, and crusting than the lesions of pityriasis rosea.

Guttate psoriasis, which occurs following streptococcal pharyngitis in over 50% of patients, does not present with a herald lesion or distribution along Langer’s lines.10 If guttate psoriasis is suspected, rapid streptococcal testing of the throat or perianal area may be considered.

Nummular eczema presents as papules that enlarge to form erythematous, lichenified plaques that measure 1-2 cm in diameter. A relatively sudden eruption, such as this patient’s, would be unusual for nummular eczema. Also, nummular eczema typically occurs on xerotic skin, more often on the extremities than the trunk.
 

Diagnostic tests, treatment

Dr. Catalina Matiz, a pediatric dermatologist at Southern California Permanente Medical Group, San Diego
Dr. Catalina Matiz
Pityriasis rosea is diagnosed clinically, although if the clinical picture is not clear, tests for tinea corporis (potassium hydroxide test, fungal culture) and syphilis ( rapid plasma reagin test, venereal disease research laboratory test, fluorescent treponemal antibody–absorption test) can be performed to exclude these possibilities. Skin biopsy is rarely performed because histologic findings usually are not specific and include focal parakeratosis, spongiosis, acanthosis, and perivascular lymphohistiocytic infiltrate.

Most patients do not require specific therapy for pityriasis rosea. Patients should be reassured that PR is typically a self-limited disease without long-term sequelae. Pregnant patients who develop pityriasis rosea in the first trimester may be at higher risk for spontaneous abortion,although data on the subject are sorely lacking.11 Oral antihistamines are useful in reducing pruritus associated with PR, and some patients experience relief by applying a low-potency topical corticosteroid.

In more severe cases, or in cases in which the patient is greatly distressed by the lesions, both broadband and narrowband UVB phototherapy effectively improve severity of lesions and reduces symptoms.12 These observations suggest that moderate sun exposure can help to reduce severity of PR lesions and hasten their resolution, but no studies assessing the effect of sun exposure on pityriasis rosea symptoms have been performed.

Furthermore, the possible role of the HHV-6 in PR has led some investigators to explore the utility of acyclovir in managing pityriasis rosea.13 One group recently found that 400 mg of acyclovir three times per day for 7 days decreased the number of lesions and pruritus associated with pityriasis rosea, compared those seen in controls, at 1-month follow-up.13

Pityriasis rosea is a self-limited and benign condition, but with its rapid onset and striking appearance, can be distressing to patients. Timely recognition of the diagnosis, consideration of mimics, and ample reassurance are appropriate when approaching this disease.
 

Mr. Kusari is with the division of pediatric and adolescent dermatology at Rady Children’s Hospital, San Diego, and the departments of dermatology and pediatrics, University of California, San Diego. Dr. Matiz is a pediatric dermatologist at Southern California Permanente Medical Group, San Diego. They have no relevant financial disclosures. Email them at pdnews@frontlinemedcom.com.

References

1. Dermatology. 2015;231(1):9-14.

2. World J Clin Cases. 2017 Jun 16;5(6):203-11.

3. Pediatr Dermatol. 2011 May-Jun;28(3):341-2.

4. J Eur Acad Dermatol Venereol. 2006 Jul;20(6):667-71.

5. Dermatology. 1997;195(4):374-8.

6. J Invest Dermatol. 2005 Jun;124(6):1234-40.

7. Arch Dermatol. 1999 Sep;135(9):1070-2.

8. J Am Acad Dermatol. 1982 Jul;7(1):80-9.

9. Iran J Pediatr. 2010 Jun;20(2):237-41.

10. J Pediatr. 1988 Dec;113(6):1037-9.

11. J Am Acad Dermatol. 2008 May;58(5 Suppl 1):S78-83.

12. J Am Acad Dermatol. 1995 Dec;33(6):996-9.

13. Indian Dermatol Online J. 2015 May-Jun;6(3):181-4.

Publications
Topics
Sections

 

The patient was diagnosed with pityriasis rosea (PR) on the basis of the clinical findings; a biopsy was not performed. The patient’s pruritus was treated with oral hydroxyzine and topical 1% triamcinolone ointment. She experienced itch relief with these treatments. On follow-up at 3 months, the patient’s lesions had mostly resolved with some postinflammatory hyperpigmentation.

Multiple scaly, pink, oval plaques and papules on the 6-year-old female patient's torso are pityriasis rosea lesions.
Courtesy Dr. Catalina Matiz
First described as “roseola annulata” by the English physician Robert Willan in 1798, pityriasis rosea is a benign, self-limited condition whose precise etiology remains unknown. The disease is typically characterized by the eruption of a large, scaly, pink- to salmon-colored “herald patch” followed by the eruption of smaller plaques over a matter of hours or days. Up to 40% of cases may not present with a herald patch. In children, the time lapse between the appearance of the herald patch, when it is present, and the generalized eruption is shorter, about 4 days on average, compared with 14 days in adults.1 The eruption in pediatric PR also resolves more quickly in children – on average, within 16 days – compared with 45 days in adults.1 Most patients experience complete resolution of the lesions within 3 months.

In some patients, flu-like symptoms precede the onset of skin lesions; this has led to speculation regarding a viral etiology for PR. This prodrome, which is present in as many as half of all cases,can include mild headache, low-grade fever, joint aches, or malaise.2 Pityriasis rosea is thought to occur secondary to a systemic activation of human herpesviruses (HHV) 6 and/or HHV-7. Three cases of PR have been reported in the setting of H1N1 influenza virus infection.3 In one small study, HHV-8 was detected by polymerase chain reaction in approximately 20% of biopsy samples of lesional skin in patients with PR.4 However, most research on a viral etiology for pityriasis rosea has focused on HHV-6 and to a lesser extent HHV-7. DNA from both viruses has been isolated from PR lesions, but at varying detection rates.5,6 Furthermore, HHV-7 DNA has been isolated in as many as 14% of normal individuals without pityriasis rosea, suggesting that the presence of this virus on the skin is fairly common.7

Pityriasis rosea occurs in males and females of all ethnicities, with a slight female predominance. It is rare in young children and older adults. Most cases occur in adolescents and in adults in their twenties and early thirties. Cases occur most frequently in fall and spring.8

The herald patch of pityriasis rosea is typically solitary, but cases with multiple herald patches have been described. The herald patch can range in size from 1-10 cm and usually contains the best example of trailing scale – scale seen on the inside edge of the annular lesion. The satellite lesions of pityriasis rosea are typically papules or plaques with a collarette of scale. These lesions usually are oriented along the Langer cleavage lines, giving them a “Christmas tree” configuration when they appear on the posterior trunk.

Ayan Kusari, University of California, San Francisco
Ayan Kusari
PR usually presents on the trunk rather than on the extremities, although cases in which there is greater involvement of the extremities, axillae, and groin have been described.9 Such cases are referred to as pityriasis rosea inversa and account for fewer than 5% of all cases.9 Other unusual presentations of pityriasis rosea, including purpura, hemorrhagic lesions, targetoid lesions, vesicles, and urticated plaques have been described. In such cases, biopsy can be useful, more to exclude other diagnoses than to definitively diagnose pityriasis rosea. Fewer than 20% of patients may develop oral lesions.
 

Mimics

The herald patch of pityriasis rosea can resemble tinea corporis, and if there is any doubt as to the diagnosis, potassium hydroxide examination (also known as a KOH test) and/or fungal culture should be done to rule out a fungal etiology. However, certain features of this case, particularly the subsequent development of satellite lesions, are more consistent with pityriasis rosea.

Secondary syphilis should be considered in patients who are sexually active. The lesions of secondary syphilis are not typically pruritic, and involvement of the palms and soles is common (whereas such involvement is rare in pityriasis rosea).

Like pityriasis rosea, pityriasis lichenoides et varioliformis acuta (PLEVA) is characterized by papular lesions that resolve spontaneously; the lesions of PLEVA usually evolve to vesicular, necrotic, and purpuric papules that take longer to resolve than PR lesions. The lesions of PLEVA are more erythematous, pustular, and crusting than the lesions of pityriasis rosea.

Guttate psoriasis, which occurs following streptococcal pharyngitis in over 50% of patients, does not present with a herald lesion or distribution along Langer’s lines.10 If guttate psoriasis is suspected, rapid streptococcal testing of the throat or perianal area may be considered.

Nummular eczema presents as papules that enlarge to form erythematous, lichenified plaques that measure 1-2 cm in diameter. A relatively sudden eruption, such as this patient’s, would be unusual for nummular eczema. Also, nummular eczema typically occurs on xerotic skin, more often on the extremities than the trunk.
 

Diagnostic tests, treatment

Dr. Catalina Matiz, a pediatric dermatologist at Southern California Permanente Medical Group, San Diego
Dr. Catalina Matiz
Pityriasis rosea is diagnosed clinically, although if the clinical picture is not clear, tests for tinea corporis (potassium hydroxide test, fungal culture) and syphilis ( rapid plasma reagin test, venereal disease research laboratory test, fluorescent treponemal antibody–absorption test) can be performed to exclude these possibilities. Skin biopsy is rarely performed because histologic findings usually are not specific and include focal parakeratosis, spongiosis, acanthosis, and perivascular lymphohistiocytic infiltrate.

Most patients do not require specific therapy for pityriasis rosea. Patients should be reassured that PR is typically a self-limited disease without long-term sequelae. Pregnant patients who develop pityriasis rosea in the first trimester may be at higher risk for spontaneous abortion,although data on the subject are sorely lacking.11 Oral antihistamines are useful in reducing pruritus associated with PR, and some patients experience relief by applying a low-potency topical corticosteroid.

In more severe cases, or in cases in which the patient is greatly distressed by the lesions, both broadband and narrowband UVB phototherapy effectively improve severity of lesions and reduces symptoms.12 These observations suggest that moderate sun exposure can help to reduce severity of PR lesions and hasten their resolution, but no studies assessing the effect of sun exposure on pityriasis rosea symptoms have been performed.

Furthermore, the possible role of the HHV-6 in PR has led some investigators to explore the utility of acyclovir in managing pityriasis rosea.13 One group recently found that 400 mg of acyclovir three times per day for 7 days decreased the number of lesions and pruritus associated with pityriasis rosea, compared those seen in controls, at 1-month follow-up.13

Pityriasis rosea is a self-limited and benign condition, but with its rapid onset and striking appearance, can be distressing to patients. Timely recognition of the diagnosis, consideration of mimics, and ample reassurance are appropriate when approaching this disease.
 

Mr. Kusari is with the division of pediatric and adolescent dermatology at Rady Children’s Hospital, San Diego, and the departments of dermatology and pediatrics, University of California, San Diego. Dr. Matiz is a pediatric dermatologist at Southern California Permanente Medical Group, San Diego. They have no relevant financial disclosures. Email them at pdnews@frontlinemedcom.com.

References

1. Dermatology. 2015;231(1):9-14.

2. World J Clin Cases. 2017 Jun 16;5(6):203-11.

3. Pediatr Dermatol. 2011 May-Jun;28(3):341-2.

4. J Eur Acad Dermatol Venereol. 2006 Jul;20(6):667-71.

5. Dermatology. 1997;195(4):374-8.

6. J Invest Dermatol. 2005 Jun;124(6):1234-40.

7. Arch Dermatol. 1999 Sep;135(9):1070-2.

8. J Am Acad Dermatol. 1982 Jul;7(1):80-9.

9. Iran J Pediatr. 2010 Jun;20(2):237-41.

10. J Pediatr. 1988 Dec;113(6):1037-9.

11. J Am Acad Dermatol. 2008 May;58(5 Suppl 1):S78-83.

12. J Am Acad Dermatol. 1995 Dec;33(6):996-9.

13. Indian Dermatol Online J. 2015 May-Jun;6(3):181-4.

 

The patient was diagnosed with pityriasis rosea (PR) on the basis of the clinical findings; a biopsy was not performed. The patient’s pruritus was treated with oral hydroxyzine and topical 1% triamcinolone ointment. She experienced itch relief with these treatments. On follow-up at 3 months, the patient’s lesions had mostly resolved with some postinflammatory hyperpigmentation.

Multiple scaly, pink, oval plaques and papules on the 6-year-old female patient's torso are pityriasis rosea lesions.
Courtesy Dr. Catalina Matiz
First described as “roseola annulata” by the English physician Robert Willan in 1798, pityriasis rosea is a benign, self-limited condition whose precise etiology remains unknown. The disease is typically characterized by the eruption of a large, scaly, pink- to salmon-colored “herald patch” followed by the eruption of smaller plaques over a matter of hours or days. Up to 40% of cases may not present with a herald patch. In children, the time lapse between the appearance of the herald patch, when it is present, and the generalized eruption is shorter, about 4 days on average, compared with 14 days in adults.1 The eruption in pediatric PR also resolves more quickly in children – on average, within 16 days – compared with 45 days in adults.1 Most patients experience complete resolution of the lesions within 3 months.

In some patients, flu-like symptoms precede the onset of skin lesions; this has led to speculation regarding a viral etiology for PR. This prodrome, which is present in as many as half of all cases,can include mild headache, low-grade fever, joint aches, or malaise.2 Pityriasis rosea is thought to occur secondary to a systemic activation of human herpesviruses (HHV) 6 and/or HHV-7. Three cases of PR have been reported in the setting of H1N1 influenza virus infection.3 In one small study, HHV-8 was detected by polymerase chain reaction in approximately 20% of biopsy samples of lesional skin in patients with PR.4 However, most research on a viral etiology for pityriasis rosea has focused on HHV-6 and to a lesser extent HHV-7. DNA from both viruses has been isolated from PR lesions, but at varying detection rates.5,6 Furthermore, HHV-7 DNA has been isolated in as many as 14% of normal individuals without pityriasis rosea, suggesting that the presence of this virus on the skin is fairly common.7

Pityriasis rosea occurs in males and females of all ethnicities, with a slight female predominance. It is rare in young children and older adults. Most cases occur in adolescents and in adults in their twenties and early thirties. Cases occur most frequently in fall and spring.8

The herald patch of pityriasis rosea is typically solitary, but cases with multiple herald patches have been described. The herald patch can range in size from 1-10 cm and usually contains the best example of trailing scale – scale seen on the inside edge of the annular lesion. The satellite lesions of pityriasis rosea are typically papules or plaques with a collarette of scale. These lesions usually are oriented along the Langer cleavage lines, giving them a “Christmas tree” configuration when they appear on the posterior trunk.

Ayan Kusari, University of California, San Francisco
Ayan Kusari
PR usually presents on the trunk rather than on the extremities, although cases in which there is greater involvement of the extremities, axillae, and groin have been described.9 Such cases are referred to as pityriasis rosea inversa and account for fewer than 5% of all cases.9 Other unusual presentations of pityriasis rosea, including purpura, hemorrhagic lesions, targetoid lesions, vesicles, and urticated plaques have been described. In such cases, biopsy can be useful, more to exclude other diagnoses than to definitively diagnose pityriasis rosea. Fewer than 20% of patients may develop oral lesions.
 

Mimics

The herald patch of pityriasis rosea can resemble tinea corporis, and if there is any doubt as to the diagnosis, potassium hydroxide examination (also known as a KOH test) and/or fungal culture should be done to rule out a fungal etiology. However, certain features of this case, particularly the subsequent development of satellite lesions, are more consistent with pityriasis rosea.

Secondary syphilis should be considered in patients who are sexually active. The lesions of secondary syphilis are not typically pruritic, and involvement of the palms and soles is common (whereas such involvement is rare in pityriasis rosea).

Like pityriasis rosea, pityriasis lichenoides et varioliformis acuta (PLEVA) is characterized by papular lesions that resolve spontaneously; the lesions of PLEVA usually evolve to vesicular, necrotic, and purpuric papules that take longer to resolve than PR lesions. The lesions of PLEVA are more erythematous, pustular, and crusting than the lesions of pityriasis rosea.

Guttate psoriasis, which occurs following streptococcal pharyngitis in over 50% of patients, does not present with a herald lesion or distribution along Langer’s lines.10 If guttate psoriasis is suspected, rapid streptococcal testing of the throat or perianal area may be considered.

Nummular eczema presents as papules that enlarge to form erythematous, lichenified plaques that measure 1-2 cm in diameter. A relatively sudden eruption, such as this patient’s, would be unusual for nummular eczema. Also, nummular eczema typically occurs on xerotic skin, more often on the extremities than the trunk.
 

Diagnostic tests, treatment

Dr. Catalina Matiz, a pediatric dermatologist at Southern California Permanente Medical Group, San Diego
Dr. Catalina Matiz
Pityriasis rosea is diagnosed clinically, although if the clinical picture is not clear, tests for tinea corporis (potassium hydroxide test, fungal culture) and syphilis ( rapid plasma reagin test, venereal disease research laboratory test, fluorescent treponemal antibody–absorption test) can be performed to exclude these possibilities. Skin biopsy is rarely performed because histologic findings usually are not specific and include focal parakeratosis, spongiosis, acanthosis, and perivascular lymphohistiocytic infiltrate.

Most patients do not require specific therapy for pityriasis rosea. Patients should be reassured that PR is typically a self-limited disease without long-term sequelae. Pregnant patients who develop pityriasis rosea in the first trimester may be at higher risk for spontaneous abortion,although data on the subject are sorely lacking.11 Oral antihistamines are useful in reducing pruritus associated with PR, and some patients experience relief by applying a low-potency topical corticosteroid.

In more severe cases, or in cases in which the patient is greatly distressed by the lesions, both broadband and narrowband UVB phototherapy effectively improve severity of lesions and reduces symptoms.12 These observations suggest that moderate sun exposure can help to reduce severity of PR lesions and hasten their resolution, but no studies assessing the effect of sun exposure on pityriasis rosea symptoms have been performed.

Furthermore, the possible role of the HHV-6 in PR has led some investigators to explore the utility of acyclovir in managing pityriasis rosea.13 One group recently found that 400 mg of acyclovir three times per day for 7 days decreased the number of lesions and pruritus associated with pityriasis rosea, compared those seen in controls, at 1-month follow-up.13

Pityriasis rosea is a self-limited and benign condition, but with its rapid onset and striking appearance, can be distressing to patients. Timely recognition of the diagnosis, consideration of mimics, and ample reassurance are appropriate when approaching this disease.
 

Mr. Kusari is with the division of pediatric and adolescent dermatology at Rady Children’s Hospital, San Diego, and the departments of dermatology and pediatrics, University of California, San Diego. Dr. Matiz is a pediatric dermatologist at Southern California Permanente Medical Group, San Diego. They have no relevant financial disclosures. Email them at pdnews@frontlinemedcom.com.

References

1. Dermatology. 2015;231(1):9-14.

2. World J Clin Cases. 2017 Jun 16;5(6):203-11.

3. Pediatr Dermatol. 2011 May-Jun;28(3):341-2.

4. J Eur Acad Dermatol Venereol. 2006 Jul;20(6):667-71.

5. Dermatology. 1997;195(4):374-8.

6. J Invest Dermatol. 2005 Jun;124(6):1234-40.

7. Arch Dermatol. 1999 Sep;135(9):1070-2.

8. J Am Acad Dermatol. 1982 Jul;7(1):80-9.

9. Iran J Pediatr. 2010 Jun;20(2):237-41.

10. J Pediatr. 1988 Dec;113(6):1037-9.

11. J Am Acad Dermatol. 2008 May;58(5 Suppl 1):S78-83.

12. J Am Acad Dermatol. 1995 Dec;33(6):996-9.

13. Indian Dermatol Online J. 2015 May-Jun;6(3):181-4.

Publications
Publications
Topics
Article Type
Sections
Questionnaire Body

A 6-year-old female presents to the pediatric dermatology office with a 2-day history of a slightly itchy skin lesion on her back. Her birthday was a week prior, and her mother gave her a new kitten, and since then she has been playing with the kitten daily. She has tried some over-the-counter antifungal cream since the lesion first appeared, but there hasn’t been much improvement. The night prior to presenting to the office, the mother noticed more lesions developing on the child’s torso, and because of this, she became worried.

Multiple scaly, pink, oval plaques and papules on the 6-year-old female patient's torso are pityriasis rosea lesions.
Courtesy Dr. Catalina Matiz
The girl denies any other symptoms. She has a history of an upper respiratory infection and had a sore throat 2 months prior, which resolved without medical treatment. The mother reports there are no other family members affected. The patient has two other siblings, who also have been playing with the cat but are asymptomatic. The father has a history of psoriasis, and the mother had a history of atopic dermatitis as a child.

On physical exam, the patient is well appearing, and vital signs are normal. She has multiple scaly, pink, oval plaques and papules on her torso. There are no oral lesions, and her palms and soles are spared.

Disallow All Ads
Content Gating
No Gating (article Unlocked/Free)
Alternative CME
Disqus Comments
Default

Pediatric Dermatology Consult - November 2017

Article Type
Changed
Fri, 01/18/2019 - 17:07
Display Headline
Pediatric Dermatology Consult - November 2017

 

The patient was diagnosed with Henoch-Schönlein purpura (HSP) based on clinical presentation of the lesions and associated symptoms of arthralgia and abdominal pain. Urinalysis was obtained and found to be unremarkable, at presentation and follow-up, and treatment with naproxen 5 mg/kg divided into two doses per day was started for pain relief. A prednisone taper starting at 1 mg/kg per day for 3 weeks also was started due to the presence of severe abdominal pain and bullae on exam. The patient was followed with regular urine studies and blood pressure checks for 2 months, and these also were within normal limits.

HSP, also known as anaphylactoid purpura and immunoglobulin A (IgA) vasculitis, is a small vessel leukocytoclastic vasculitis characterized by the perivascular deposition of IgA1-based immune complexes in the walls of arterioles and postcapillary venules.1The clinical picture of HSP is palpable purpura, abdominal pain, arthritis, and hematuria 1-2 weeks following an upper respiratory infection. In the vast majority of cases, the condition resolves spontaneously in 4-6 weeks and does not require any specific treatment,2 although NSAIDs and systemic corticosteroids can be used for mild-to-moderate and severe pain, respectively.3

HSP is the most common vasculitis in children, with a peak incidence in boys under the age of 5 years. It occurs worldwide, more commonly among whites and Asians, less commonly among blacks, and recent studies from the Czech Republic,4 Taiwan,5 Spain,6 France,7 South Korea,8 and the United Kingdom9 have shown similar incidence rates of 10-20 per 100,000 children. HSP does occur in adults, but is less common, and is known to carry a worse prognosis – in particular, a higher risk of progression to chronic kidney disease. The disease is more commonly seen in winter months,1 unsurprisingly as upper respiratory tract infections also are more common in these months.10
 

Pathogenesis

The exact pathogenesis of HSP is the subject of ongoing investigation and continued controversy. Mutations and polymorphisms in mannose-binding lectin, interleukins 1 and 8, vascular endothelial growth factor, and alpha-1-antitrypsin have been associated with HSP.3 Immunoglobulin A (IgA) normally exists in two heavily glycosylated forms – IgA1 and IgA2. Abnormal glycosylation, particularly undergalactosylation, of IgA1, the predominant form of IgA in serum and mucosal secretions, has been linked to HSP.11 HSP has been associated with group A streptococcal infections, Bartonella henselae (cat scratch fever) and numerous drugs,12 although no definitive causal or mechanistic explanation has been identified.

Diagnosis

Two major diagnostic criteria for HSP are widely in use, one developed by the American College of Rheumatology (ACR) in 199013 and the other by the European League Against Rheumatism (EULAR) in 2005.14 Both the ACR and EULAR criteria include acute abdominal pain, purpura, and microscopic evidence of vasculitis. Almost all patients with HSP have cutaneous purpura, and many of these patients have palpable purpura, which is pathognomonic of a leukocytoclastic vasculitis, but palpable purpura is not needed for diagnosis. The ACR criteria additionally include age of 20 years or younger, while the EULAR criteria include arthralgias and the presence of hematuria or proteinuria. Ancillary testing usually is not required to make the diagnosis, but when the diagnosis is not clear histopathologic analysis of a skin sample can identify leukocytoclastic vasculitis. Other laboratory studies that may be needed to rule out other conditions, as well as other organ involvement, include a complete blood count, which can be done to rule out thrombocytopenia as a cause of purpura, a metabolic panel, coagulation studies, occult blood test of stool, abdominal imaging, and urinalysis (UA), which can identify proteinuria or hematuria.

Abdominal pain in HSP is believed to be a result of vasculitis of the gastric, mesenteric, and/or colic vasculature. Bleeding from the inflamed vasculature rarely can lead to gross hematochezia, frank melena, or hematemesis. One serious, potential complication of HSP-related mesenteric vasculitis is intussusception, which is otherwise rare in children older than 2 years. Intussusception should be suspected if features of the classic triad of episodic abdominal pain, sausage-shaped abdominal mass, and currant jelly stool are present. Abdominal ultrasound can help to determine whether intussusception is present.

The purpura in HSP presents in waves or crops, and crops last 5-10 days each. Complete resolution takes 4-6 weeks. If biopsy is desired to confirm the diagnosis, it should be done on a lesion less than 24 hours old. This allows for identification of perivascular IgA on histopathology: beyond 24 hours, IgG and IgM also leak out, contributing to a less specific histopathologic picture.

Accurate diagnosis of HSP is important to guide therapy and anticipate potential complications. Wegener’s granulomatosis (A), also known as granulomatosis with polyangiitis, classically involves the upper and lower respiratory tract and the kidneys, leading to a presentation of epistaxis, cough, and hypertension. It occurs more commonly in adults than children. Finkelstein disease (B), also known as acute hemorrhagic edema of infancy (AHEI), is characterized by the development of petechial, urticarial, or targetoid plaques over 24-48 hours with tender edema and fever in children aged less than 2 years. Unlike HSP, AHEI typically does not involve the gastrointestinal tract, kidneys, or joints. Biopsy of skin lesions of AHEI reveals IgA deposition and leukocytoclastic vasculitis, leading some authors to consider it a closely related entity to HSP. Microscopic polyangiitis (D) is an uncommon pauci-immune vasculitis similar to Wegener’s granulomatosis, but lacking granulomas. It presents typically in the 5th decade of life with fever, fatigue, weight loss, and renal involvement. IgA nephropathy (E), also known as synpharyngitic nephritis and Berger disease, is less likely than HSP to cause a rash, joint pain, or abdominal pain. The nomenclature of HSP (whose alternate name is IgA vasculitis) reflects the multi-organ nature of HSP in comparison to IgA nephropathy, which is more likely to be limited to the kidneys.
 

Treatment

Aside from intussusception and renal disease, which may result from HSP, treatment is not typically required for HSP as it resolves spontaneously. Patients with significant arthralgias are likely to benefit from NSAIDs such as naproxen 5-20 mg/kg per day, although NSAIDs should be avoided if there is significant renal dysfunction or GI bleeding. Patients with severe abdominal pain or joint pain may be more likely to benefit from oral corticosteroids, particularly prednisone 1-2 mg/kg per day. A meta-analysis showed that corticosteroids significantly reduce the duration of symptoms if given early in the course of disease.15

The prognosis is usually excellent, except for a very small sample of the population (5%) that can develop end-stage renal disease. It is recommended that all children with HSP continue monitoring blood pressure and UA either weekly or biweekly for the first 2 months and then once a month for 6-12 months.16

First described in 1801 by a British physician, HSP is a common and usually self-limited disease for which our understanding has advanced greatly over the past 2 centuries, yet for which many important questions regarding pathophysiology remain unanswered. No diagnostic tests or treatments are needed for the majority of patients. Providers should include HSP in the differential diagnosis for the child with unexplained abdominal pain, renal dysfunction, or nonthrombocytopenic purpura.
 

Mr. Kusari is a medical student at the University of California, San Diego. Dr. Matiz is a practicing dermatologist at Southern California Permanente Medical Group in La Mesa, California. Dr. Matiz and Mr. Kusari said they had no relevant financial disclosures. Email them at pdnews@frontlinemedcom.com.

References

1. “Hurwitz Clinical Pediatric Dermatology: A Textbook of Skin Disorders of Childhood and Adolescence”, 5th ed. (New York: Elsevier, 2016).

2. Lancet. 2007;369(9566):976-8.

3. “Dermatology”, 3rd ed. (Philadelphia: Elsevier Saunders, 2012).

4. J Rheumatol. 2004 Nov;31(11):2295-9.

5. Rheumatology (Oxford). 2005 May;44(5):618-22.

6. Medicine (Baltimore). 2014 Mar;93(2):106-13.

7. Rheumatology (Oxford). 2017;56(8):1358-66.

8. J Korean Med Sci. 2014 Feb;29(2):198-203.

9. Lancet. 2002 Oct 19;360(9341):1197-202.

10. Rhinology. 2015 Jun;53(2):99-106.

11. PLoS One. 2016 Nov 21;11(11):e0166700.

12. Pediatr Infect Dis J. 2002 Jan;21(1):28-31.

13. Arthritis Rheum. 1990 Aug;33(8):1114-21.

14. Ann Rheum Dis. 2006 Jul;65(7):936-41.

15. Pediatrics. 2007 Nov;120(5):1079-87.

16. Arch Dis Child. 2010 Nov;95(11):877-82.

Publications
Topics
Sections

 

The patient was diagnosed with Henoch-Schönlein purpura (HSP) based on clinical presentation of the lesions and associated symptoms of arthralgia and abdominal pain. Urinalysis was obtained and found to be unremarkable, at presentation and follow-up, and treatment with naproxen 5 mg/kg divided into two doses per day was started for pain relief. A prednisone taper starting at 1 mg/kg per day for 3 weeks also was started due to the presence of severe abdominal pain and bullae on exam. The patient was followed with regular urine studies and blood pressure checks for 2 months, and these also were within normal limits.

HSP, also known as anaphylactoid purpura and immunoglobulin A (IgA) vasculitis, is a small vessel leukocytoclastic vasculitis characterized by the perivascular deposition of IgA1-based immune complexes in the walls of arterioles and postcapillary venules.1The clinical picture of HSP is palpable purpura, abdominal pain, arthritis, and hematuria 1-2 weeks following an upper respiratory infection. In the vast majority of cases, the condition resolves spontaneously in 4-6 weeks and does not require any specific treatment,2 although NSAIDs and systemic corticosteroids can be used for mild-to-moderate and severe pain, respectively.3

HSP is the most common vasculitis in children, with a peak incidence in boys under the age of 5 years. It occurs worldwide, more commonly among whites and Asians, less commonly among blacks, and recent studies from the Czech Republic,4 Taiwan,5 Spain,6 France,7 South Korea,8 and the United Kingdom9 have shown similar incidence rates of 10-20 per 100,000 children. HSP does occur in adults, but is less common, and is known to carry a worse prognosis – in particular, a higher risk of progression to chronic kidney disease. The disease is more commonly seen in winter months,1 unsurprisingly as upper respiratory tract infections also are more common in these months.10
 

Pathogenesis

The exact pathogenesis of HSP is the subject of ongoing investigation and continued controversy. Mutations and polymorphisms in mannose-binding lectin, interleukins 1 and 8, vascular endothelial growth factor, and alpha-1-antitrypsin have been associated with HSP.3 Immunoglobulin A (IgA) normally exists in two heavily glycosylated forms – IgA1 and IgA2. Abnormal glycosylation, particularly undergalactosylation, of IgA1, the predominant form of IgA in serum and mucosal secretions, has been linked to HSP.11 HSP has been associated with group A streptococcal infections, Bartonella henselae (cat scratch fever) and numerous drugs,12 although no definitive causal or mechanistic explanation has been identified.

Diagnosis

Two major diagnostic criteria for HSP are widely in use, one developed by the American College of Rheumatology (ACR) in 199013 and the other by the European League Against Rheumatism (EULAR) in 2005.14 Both the ACR and EULAR criteria include acute abdominal pain, purpura, and microscopic evidence of vasculitis. Almost all patients with HSP have cutaneous purpura, and many of these patients have palpable purpura, which is pathognomonic of a leukocytoclastic vasculitis, but palpable purpura is not needed for diagnosis. The ACR criteria additionally include age of 20 years or younger, while the EULAR criteria include arthralgias and the presence of hematuria or proteinuria. Ancillary testing usually is not required to make the diagnosis, but when the diagnosis is not clear histopathologic analysis of a skin sample can identify leukocytoclastic vasculitis. Other laboratory studies that may be needed to rule out other conditions, as well as other organ involvement, include a complete blood count, which can be done to rule out thrombocytopenia as a cause of purpura, a metabolic panel, coagulation studies, occult blood test of stool, abdominal imaging, and urinalysis (UA), which can identify proteinuria or hematuria.

Abdominal pain in HSP is believed to be a result of vasculitis of the gastric, mesenteric, and/or colic vasculature. Bleeding from the inflamed vasculature rarely can lead to gross hematochezia, frank melena, or hematemesis. One serious, potential complication of HSP-related mesenteric vasculitis is intussusception, which is otherwise rare in children older than 2 years. Intussusception should be suspected if features of the classic triad of episodic abdominal pain, sausage-shaped abdominal mass, and currant jelly stool are present. Abdominal ultrasound can help to determine whether intussusception is present.

The purpura in HSP presents in waves or crops, and crops last 5-10 days each. Complete resolution takes 4-6 weeks. If biopsy is desired to confirm the diagnosis, it should be done on a lesion less than 24 hours old. This allows for identification of perivascular IgA on histopathology: beyond 24 hours, IgG and IgM also leak out, contributing to a less specific histopathologic picture.

Accurate diagnosis of HSP is important to guide therapy and anticipate potential complications. Wegener’s granulomatosis (A), also known as granulomatosis with polyangiitis, classically involves the upper and lower respiratory tract and the kidneys, leading to a presentation of epistaxis, cough, and hypertension. It occurs more commonly in adults than children. Finkelstein disease (B), also known as acute hemorrhagic edema of infancy (AHEI), is characterized by the development of petechial, urticarial, or targetoid plaques over 24-48 hours with tender edema and fever in children aged less than 2 years. Unlike HSP, AHEI typically does not involve the gastrointestinal tract, kidneys, or joints. Biopsy of skin lesions of AHEI reveals IgA deposition and leukocytoclastic vasculitis, leading some authors to consider it a closely related entity to HSP. Microscopic polyangiitis (D) is an uncommon pauci-immune vasculitis similar to Wegener’s granulomatosis, but lacking granulomas. It presents typically in the 5th decade of life with fever, fatigue, weight loss, and renal involvement. IgA nephropathy (E), also known as synpharyngitic nephritis and Berger disease, is less likely than HSP to cause a rash, joint pain, or abdominal pain. The nomenclature of HSP (whose alternate name is IgA vasculitis) reflects the multi-organ nature of HSP in comparison to IgA nephropathy, which is more likely to be limited to the kidneys.
 

Treatment

Aside from intussusception and renal disease, which may result from HSP, treatment is not typically required for HSP as it resolves spontaneously. Patients with significant arthralgias are likely to benefit from NSAIDs such as naproxen 5-20 mg/kg per day, although NSAIDs should be avoided if there is significant renal dysfunction or GI bleeding. Patients with severe abdominal pain or joint pain may be more likely to benefit from oral corticosteroids, particularly prednisone 1-2 mg/kg per day. A meta-analysis showed that corticosteroids significantly reduce the duration of symptoms if given early in the course of disease.15

The prognosis is usually excellent, except for a very small sample of the population (5%) that can develop end-stage renal disease. It is recommended that all children with HSP continue monitoring blood pressure and UA either weekly or biweekly for the first 2 months and then once a month for 6-12 months.16

First described in 1801 by a British physician, HSP is a common and usually self-limited disease for which our understanding has advanced greatly over the past 2 centuries, yet for which many important questions regarding pathophysiology remain unanswered. No diagnostic tests or treatments are needed for the majority of patients. Providers should include HSP in the differential diagnosis for the child with unexplained abdominal pain, renal dysfunction, or nonthrombocytopenic purpura.
 

Mr. Kusari is a medical student at the University of California, San Diego. Dr. Matiz is a practicing dermatologist at Southern California Permanente Medical Group in La Mesa, California. Dr. Matiz and Mr. Kusari said they had no relevant financial disclosures. Email them at pdnews@frontlinemedcom.com.

References

1. “Hurwitz Clinical Pediatric Dermatology: A Textbook of Skin Disorders of Childhood and Adolescence”, 5th ed. (New York: Elsevier, 2016).

2. Lancet. 2007;369(9566):976-8.

3. “Dermatology”, 3rd ed. (Philadelphia: Elsevier Saunders, 2012).

4. J Rheumatol. 2004 Nov;31(11):2295-9.

5. Rheumatology (Oxford). 2005 May;44(5):618-22.

6. Medicine (Baltimore). 2014 Mar;93(2):106-13.

7. Rheumatology (Oxford). 2017;56(8):1358-66.

8. J Korean Med Sci. 2014 Feb;29(2):198-203.

9. Lancet. 2002 Oct 19;360(9341):1197-202.

10. Rhinology. 2015 Jun;53(2):99-106.

11. PLoS One. 2016 Nov 21;11(11):e0166700.

12. Pediatr Infect Dis J. 2002 Jan;21(1):28-31.

13. Arthritis Rheum. 1990 Aug;33(8):1114-21.

14. Ann Rheum Dis. 2006 Jul;65(7):936-41.

15. Pediatrics. 2007 Nov;120(5):1079-87.

16. Arch Dis Child. 2010 Nov;95(11):877-82.

 

The patient was diagnosed with Henoch-Schönlein purpura (HSP) based on clinical presentation of the lesions and associated symptoms of arthralgia and abdominal pain. Urinalysis was obtained and found to be unremarkable, at presentation and follow-up, and treatment with naproxen 5 mg/kg divided into two doses per day was started for pain relief. A prednisone taper starting at 1 mg/kg per day for 3 weeks also was started due to the presence of severe abdominal pain and bullae on exam. The patient was followed with regular urine studies and blood pressure checks for 2 months, and these also were within normal limits.

HSP, also known as anaphylactoid purpura and immunoglobulin A (IgA) vasculitis, is a small vessel leukocytoclastic vasculitis characterized by the perivascular deposition of IgA1-based immune complexes in the walls of arterioles and postcapillary venules.1The clinical picture of HSP is palpable purpura, abdominal pain, arthritis, and hematuria 1-2 weeks following an upper respiratory infection. In the vast majority of cases, the condition resolves spontaneously in 4-6 weeks and does not require any specific treatment,2 although NSAIDs and systemic corticosteroids can be used for mild-to-moderate and severe pain, respectively.3

HSP is the most common vasculitis in children, with a peak incidence in boys under the age of 5 years. It occurs worldwide, more commonly among whites and Asians, less commonly among blacks, and recent studies from the Czech Republic,4 Taiwan,5 Spain,6 France,7 South Korea,8 and the United Kingdom9 have shown similar incidence rates of 10-20 per 100,000 children. HSP does occur in adults, but is less common, and is known to carry a worse prognosis – in particular, a higher risk of progression to chronic kidney disease. The disease is more commonly seen in winter months,1 unsurprisingly as upper respiratory tract infections also are more common in these months.10
 

Pathogenesis

The exact pathogenesis of HSP is the subject of ongoing investigation and continued controversy. Mutations and polymorphisms in mannose-binding lectin, interleukins 1 and 8, vascular endothelial growth factor, and alpha-1-antitrypsin have been associated with HSP.3 Immunoglobulin A (IgA) normally exists in two heavily glycosylated forms – IgA1 and IgA2. Abnormal glycosylation, particularly undergalactosylation, of IgA1, the predominant form of IgA in serum and mucosal secretions, has been linked to HSP.11 HSP has been associated with group A streptococcal infections, Bartonella henselae (cat scratch fever) and numerous drugs,12 although no definitive causal or mechanistic explanation has been identified.

Diagnosis

Two major diagnostic criteria for HSP are widely in use, one developed by the American College of Rheumatology (ACR) in 199013 and the other by the European League Against Rheumatism (EULAR) in 2005.14 Both the ACR and EULAR criteria include acute abdominal pain, purpura, and microscopic evidence of vasculitis. Almost all patients with HSP have cutaneous purpura, and many of these patients have palpable purpura, which is pathognomonic of a leukocytoclastic vasculitis, but palpable purpura is not needed for diagnosis. The ACR criteria additionally include age of 20 years or younger, while the EULAR criteria include arthralgias and the presence of hematuria or proteinuria. Ancillary testing usually is not required to make the diagnosis, but when the diagnosis is not clear histopathologic analysis of a skin sample can identify leukocytoclastic vasculitis. Other laboratory studies that may be needed to rule out other conditions, as well as other organ involvement, include a complete blood count, which can be done to rule out thrombocytopenia as a cause of purpura, a metabolic panel, coagulation studies, occult blood test of stool, abdominal imaging, and urinalysis (UA), which can identify proteinuria or hematuria.

Abdominal pain in HSP is believed to be a result of vasculitis of the gastric, mesenteric, and/or colic vasculature. Bleeding from the inflamed vasculature rarely can lead to gross hematochezia, frank melena, or hematemesis. One serious, potential complication of HSP-related mesenteric vasculitis is intussusception, which is otherwise rare in children older than 2 years. Intussusception should be suspected if features of the classic triad of episodic abdominal pain, sausage-shaped abdominal mass, and currant jelly stool are present. Abdominal ultrasound can help to determine whether intussusception is present.

The purpura in HSP presents in waves or crops, and crops last 5-10 days each. Complete resolution takes 4-6 weeks. If biopsy is desired to confirm the diagnosis, it should be done on a lesion less than 24 hours old. This allows for identification of perivascular IgA on histopathology: beyond 24 hours, IgG and IgM also leak out, contributing to a less specific histopathologic picture.

Accurate diagnosis of HSP is important to guide therapy and anticipate potential complications. Wegener’s granulomatosis (A), also known as granulomatosis with polyangiitis, classically involves the upper and lower respiratory tract and the kidneys, leading to a presentation of epistaxis, cough, and hypertension. It occurs more commonly in adults than children. Finkelstein disease (B), also known as acute hemorrhagic edema of infancy (AHEI), is characterized by the development of petechial, urticarial, or targetoid plaques over 24-48 hours with tender edema and fever in children aged less than 2 years. Unlike HSP, AHEI typically does not involve the gastrointestinal tract, kidneys, or joints. Biopsy of skin lesions of AHEI reveals IgA deposition and leukocytoclastic vasculitis, leading some authors to consider it a closely related entity to HSP. Microscopic polyangiitis (D) is an uncommon pauci-immune vasculitis similar to Wegener’s granulomatosis, but lacking granulomas. It presents typically in the 5th decade of life with fever, fatigue, weight loss, and renal involvement. IgA nephropathy (E), also known as synpharyngitic nephritis and Berger disease, is less likely than HSP to cause a rash, joint pain, or abdominal pain. The nomenclature of HSP (whose alternate name is IgA vasculitis) reflects the multi-organ nature of HSP in comparison to IgA nephropathy, which is more likely to be limited to the kidneys.
 

Treatment

Aside from intussusception and renal disease, which may result from HSP, treatment is not typically required for HSP as it resolves spontaneously. Patients with significant arthralgias are likely to benefit from NSAIDs such as naproxen 5-20 mg/kg per day, although NSAIDs should be avoided if there is significant renal dysfunction or GI bleeding. Patients with severe abdominal pain or joint pain may be more likely to benefit from oral corticosteroids, particularly prednisone 1-2 mg/kg per day. A meta-analysis showed that corticosteroids significantly reduce the duration of symptoms if given early in the course of disease.15

The prognosis is usually excellent, except for a very small sample of the population (5%) that can develop end-stage renal disease. It is recommended that all children with HSP continue monitoring blood pressure and UA either weekly or biweekly for the first 2 months and then once a month for 6-12 months.16

First described in 1801 by a British physician, HSP is a common and usually self-limited disease for which our understanding has advanced greatly over the past 2 centuries, yet for which many important questions regarding pathophysiology remain unanswered. No diagnostic tests or treatments are needed for the majority of patients. Providers should include HSP in the differential diagnosis for the child with unexplained abdominal pain, renal dysfunction, or nonthrombocytopenic purpura.
 

Mr. Kusari is a medical student at the University of California, San Diego. Dr. Matiz is a practicing dermatologist at Southern California Permanente Medical Group in La Mesa, California. Dr. Matiz and Mr. Kusari said they had no relevant financial disclosures. Email them at pdnews@frontlinemedcom.com.

References

1. “Hurwitz Clinical Pediatric Dermatology: A Textbook of Skin Disorders of Childhood and Adolescence”, 5th ed. (New York: Elsevier, 2016).

2. Lancet. 2007;369(9566):976-8.

3. “Dermatology”, 3rd ed. (Philadelphia: Elsevier Saunders, 2012).

4. J Rheumatol. 2004 Nov;31(11):2295-9.

5. Rheumatology (Oxford). 2005 May;44(5):618-22.

6. Medicine (Baltimore). 2014 Mar;93(2):106-13.

7. Rheumatology (Oxford). 2017;56(8):1358-66.

8. J Korean Med Sci. 2014 Feb;29(2):198-203.

9. Lancet. 2002 Oct 19;360(9341):1197-202.

10. Rhinology. 2015 Jun;53(2):99-106.

11. PLoS One. 2016 Nov 21;11(11):e0166700.

12. Pediatr Infect Dis J. 2002 Jan;21(1):28-31.

13. Arthritis Rheum. 1990 Aug;33(8):1114-21.

14. Ann Rheum Dis. 2006 Jul;65(7):936-41.

15. Pediatrics. 2007 Nov;120(5):1079-87.

16. Arch Dis Child. 2010 Nov;95(11):877-82.

Publications
Publications
Topics
Article Type
Display Headline
Pediatric Dermatology Consult - November 2017
Display Headline
Pediatric Dermatology Consult - November 2017
Sections
Questionnaire Body

Clinical presentation


A healthy 9-year-old boy presents with 1 week history of a rash that began as “bruises” on both ankles that subsequently ascended over a few days to the proximal lower extremities and upper extremities. The rash has been painful and pruritic at times. The patient’s mother reports regular application of hydrocortisone cream for itch and pain relief, and this has been somewhat successful.

The patient has a history of longstanding constipation and abdominal pain, but over the past week has reported abdominal pain that is different and more severe than his usual abdominal pain. This abdominal pain has limited oral intake over the past 2 days. The patient and family also report bilateral pain of the wrists and elbows, which has limited his daily activities. The patient and mother deny fevers, chills, cough, coryza, and any sick contacts.

Henoch Schonlein purpura on the lower extremities in a 9 year old boy.
Catalina Matiz, MD
On physical exam, the patient is in mild distress and seems uncomfortable.

His vital signs are stable. On physical examination there is mild conjunctival injection, no intraoral lesions, and no lymphadenopathy or hepatosplenomegaly. The abdomen is not distended but it is tender to deep palpation. Bowel sounds are present. On skin examination, there are multiple purpuric annular plaques with central clearing, some with bullae and petechiae, on the bilateral buttocks and legs. There is bilateral pedal edema. On the arms, there are a few polymorphic pink and red annular to targetoid plaques.

Disallow All Ads
Content Gating
No Gating (article Unlocked/Free)
Alternative CME
Disqus Comments
Default